Torts - Missed Qs

Réussis tes devoirs et examens dès maintenant avec Quizwiz!

Under what two exceptions will the principle be vicariously liable for the Independent Contractor's actions? 2 circumstances in which Pr will be directly liable?

(i) The independent contractor is engaged in inherently dangerous activities, e.g., excavating next to a public sidewalk, blasting; or (ii) The duty, because of public policy considerations, is simply nondelegable, e.g., the duty of a business to keep its premises safe for customers. (i) Negligent selection (ii) Negligent supervision.

Under what 3 circumstances will the employer be liable for the employee's intentional torts?

(i) force is authorized in the employment; (ii) friction is generated by the employment; or (iii) the employee is furthering the business of the employer.

There often is more than one cause for an injury. The "but for" test for actual cause applies to: A Joint causes. B Alternative causes. C Superseding causes. D Concurrent causes.

Answer = D. -Concurrent Causes = But For -Joint Causes = Substantial Factor -Alternative Causes = BOP shift to each D (multiple hunters & only shot by one) -Superseding Causes = Prox-cause/after-act, unforeseeable causes. The "but for" test for actual cause applies to concurrent causes. An act or omission to act is the cause in fact of an injury when the injury would not have occurred but for the act. This test applies in concurrent cause cases, where several acts combine to cause the injury, but none of the acts standing alone would have been sufficient. But for any of the acts, the injury would not have occurred. The "substantial factor" test is used for joint causes, where several causes commingle and bring about an injury, but any one alone would have been sufficient to cause the injury. In that case, it is sufficient if defendant's conduct was a substantial factor in causing the injury. An alternative causes situation arises when two or more persons have been negligent, but uncertainty exists as to which one caused the plaintiff's injury. Under this approach, the plaintiff must prove that harm has been caused to him by one of them (with uncertainty as to which one). The burden of proof then shifts to the defendants, and each must show that his negligence is not the actual cause. Superseding causes arise in the context of proximate cause rather than actual cause. In addition to being an actual cause, the defendant's conduct must also be a proximate cause of the injury. Causes that arise after the defendant's conduct that contribute to the injury may be so unforeseeable as to be superseding causes, which cut off the defendant's liability for his original negligent act.

A defendant may be liable for a breach of the duty of care to all of the following EXCEPT: A) Rescuer employed as a firefighter or police officer B) Viable Fetus C) TP for whose economic benefit a legal or business transaction is made. D) A discovered trespasser.

Answer: A Explanation: A rescuer is a foreseeable plaintiff and is owed a duty of care as long as the rescue is not reckless. However, firefighters and police officers may be barred by the "firefighter's rule" from recovering for injuries caused by the risks of a rescue. A duty of care is owed to a viable fetus; prenatal injuries are actionable. A third party for whose economic benefit a legal or business transaction is made is owed a duty of care if the defendant could reasonably foresee harm to that party if the transaction is done negligently. While a landowner owes no duty to an undiscovered trespasser, a trespasser whose presence on the property is known to the landowner is owed a duty to be warned about seriously dangerous artificial conditions on the property.

A new homeowner had two dogs that frequently barked at birds and squirrels in the yard, especially during the day while the homeowner was at work. A neighbor who worked nights was aggravated by the barking, which disturbed his sleep, and decided to let the homeowner know how he felt. One evening, upon learning that the homeowner was entertaining her boss and several clients, the neighbor came to her front door with a boombox and started playing a recording of the dogs barking, putting it at full volume. When the homeowner came to the door, he began yelling at her and berating her in front of her guests for having no consideration for her neighbors, while continuing to play the recording. The homeowner was very upset, especially because her guests decided that they had better leave, and she ended up losing a bonus that her boss was going to give her at the end of the evening. If the homeowner asserts a claim based on intentional infliction of emotional distress against the neighbor, what will be the probable result? A The homeowner will prevail because the neighbor's conduct was extreme and outrageous. B The homeowner will prevail because she suffered pecuniary harm from the neighbor's conduct. C The neighbor will prevail because the homeowner suffered no physical harm. D The neighbor will prevail if the barking from the homeowner's dogs is judged to constitute a nuisance.

Answer: A Explanation: The homeowner will probably prevail on a claim for intentional infliction of emotional distress because the neighbor's conduct was sufficiently extreme and outrageous and the other elements of the tort are present. Intentional infliction of emotional distress requires: (i) an act by defendant amounting to extreme and outrageous conduct; (ii) intent to cause severe emotional distress or recklessness as to the effect of defendant's conduct; (iii) causation; and (iv) damages. "Outrageous conduct" is extreme conduct that transcends all bounds of decency. The neighbor's use of the recording and his insults against the homeowner for the benefit of her guests would probably qualify as extreme and outrageous conduct, particularly because there is no evidence that he had previously tried to resolve the problem with the homeowner in a more civilized manner. The neighbor had the requisite intent (either he intended to cause emotional distress or he was reckless as to its effect), there was causation, and the homeowner suffered damages (i.e., she was severely distressed) as a result of the neighbor's actions. (B) is wrong because pecuniary harm is not required for purposes of this tort—all that is required is severe emotional distress. (C) is wrong because, in contrast to negligent infliction of distress, intentional infliction of distress does not require proof of physical harm to recover. (D) is wrong because the fact that the barking constituted a nuisance would not be a defense to conduct amounting to intentional infliction of distress; abatement of a private nuisance by self-help must be preceded by notice to the other party and must be conducted in a reasonable manner.

A columnist for a major metropolitan newspaper had a very antagonistic relationship with the city's mayor. When a restaurant owned by the columnist's family was shut down by city health inspectors, the columnist responded with a column publicizing the shutdown and asserting that it was in retaliation for his prior columns in which he had criticized the mayor. In fact, the mayor had nothing to do with the action by the city health inspectors. While the columnist had no evidence of the mayor's involvement, he believed that there was a connection because "that's how the city works." Can the mayor recover against the columnist for defamation? A No, because the columnist did not act with actual malice. B No, because the columnist had a qualified privilege to explain why he believed his family's business was shut down. C Yes, because the columnist's hostility toward the mayor establishes malice so as to overcome any qualified privilege the columnist had. D Yes, because the columnist should have investigated the accuracy of his claims before publishing the column.

Answer: A Explanation: The mayor cannot recover against the columnist because he did not act with actual malice. A public official, such as a mayor, may not recover for defamatory words relating to his official conduct unless there is clear and convincing proof that the statement was made with actual malice, which is defined as knowledge that the statement was false or reckless disregard as to truth or falsity. Reckless conduct is not measured by whether a reasonable person would have investigated before publishing; rather, there must be a showing that the defendant in fact (subjectively) entertained serious doubts as to the truthfulness of his publication. Here, while the columnist had no evidence of the mayor's involvement with the action of the health inspectors, he believed that there was a connection based on his belief as to how the city operates. Hence, he has not acted with actual malice and is not liable to the mayor for defamation. (B) is incorrect because the columnist's qualified privilege applies only to statements made to defend his own actions, property, or reputation. Even if it were to apply to his explanation of why his family's restaurant was shut down, his statements in the column were beyond the scope of the privilege, which does not extend to making a statement to a mass audience whose reading of the statement would not reasonably further his interest in defending himself. Here, the publication in his newspaper column of his explanation as to why the restaurant was shut down was beyond the scope of any privilege he may have had. (C) is incorrect because malice that will result in the loss of a qualified privilege is defined by most courts as knowledge of falsity or reckless disregard as to truth or falsity, rather than hostility or ill-will. As long as the defendant is using a proper occasion for a qualified privilege in a proper way, he will not lose this privilege simply because he bears ill-will toward the plaintiff. (D) is incorrect because the fact that the columnist should have investigated the accuracy of his assertions and did not only establishes negligence on his part. As discussed above, the mayor, as a public official, must show at least reckless disregard as to truth or falsity to recover in a defamation action.

A 16-year-old teenager was playing baseball in a sandlot when the ball was hit over his head and onto a landowner's adjacent property. Ignoring "beware of dog" signs, the teenager climbed over the fence into the landowner's yard to retrieve the ball and was attacked by a vicious guard dog belonging to the landowner. The dog bit the teenager, causing him to suffer severe lacerations that required numerous stitches. If the teenager brings an action against the landowner to recover damages for his injuries, will he likely prevail? A Yes, because the landowner may not use a vicious dog to protect only his property. B Yes, because the landowner is strictly liable for injuries caused by the vicious dog. C No, because the teenager was trespassing on the landowner's property. D No, because the landowner had posted signs warning about the dog.

Answer: A Explanation: The teenager will prevail because the landowner may not intentionally use a vicious dog to protect only his property. One may use only reasonable force to defend property. A landowner may not use force that will cause death or serious bodily harm. Furthermore, one may not use indirect deadly force such as a trap, spring gun, or vicious dog when such force could not lawfully be directly used, e.g., against a mere trespasser. (B) is incorrect because strict liability in such cases generally is not imposed in favor of undiscovered trespassers against landowners. Trespassers cannot recover for injuries inflicted by the landowner's abnormally dangerous domestic animals in the absence of negligence. (C) is incorrect because a landowner who protects his property from intruders by keeping a vicious watchdog he knows is likely to cause serious bodily harm may be liable even to trespassers for injuries caused by the animal. The liability is based on intentional tort principles: Because the landowner is not entitled to use deadly force in person to protect only property, he also may not use such force indirectly. (D) is incorrect because even though the landowner posted warning signs, he can still be liable under intentional tort principles because he intentionally used the dog to protect his property, knowing that the dog is likely to cause serious bodily harm.

To fight drug abuse, a state enacted a statute forbidding the selling of model airplane glue to anyone under the age of 18 except in small quantities in prepackaged model kits. Violation of the statute was penalized by fines or, in cases of multiple violations, possible imprisonment. The statute also required that all elementary and secondary schools licensed by the state provide comprehensive drug education programs. Neither the legislature nor the courts of the state have abolished the common law tort defense of assumption of the risk. The owner of a hobby shop in the state sold a large tube of airplane glue to a 15-year-old boy who reasonably appeared to be at least 18 years old. The boy had received drug education in his school, as mandated by the statute, including coverage of the dangers of glue sniffing. The boy understood the anti-drug instruction, but he wanted to experience it for himself. The boy sniffed the glue repeatedly and suffered permanent brain damage. If the boy's parents file suit on the boy's behalf against the store owner, for whom is the court likely to rule? response - incorrect A For the boy, because the store owner violated the statute when she sold the glue to the boy. B For the store owner, because the boy is not a member of the class of persons meant to be protected by the statute. C For the store owner, because the boy was aware of the danger when he sniffed the glue. D For the store owner, because the boy could reasonably have been mistaken for an 18-year-old by the store owner.

Answer: A (Mine: D) Explanation: Strict liability crime = replacement of the statutory standard of care w/SL-standard of liability (no reasonable mistake, no AoR can apply)

A shopper was in a large department store that was remodeling its menswear department and had hired a contractor to do the work. A carpenter employed by the contractor was working on the remodeling job. When the carpenter left the store to take a break, she left a carpenter's level projecting out into one of the aisles, unbeknownst to any store employees. Shortly before she returned 15 minutes later, the shopper came down that aisle and tripped over the level. He fell and struck his head on the sharp corner of a display case. The shopper required hospitalization and sued the store for his injuries. Will the shopper prevail in his suit against the store? A Yes, because the contractor's employee left the level in the aisle. B Yes, because the store's employees had a reasonable time to discover the level before the shopper fell. C No, because the store's employees did not leave the level in the aisle. D No, because the store's employees were unaware that the level was in the aisle.

Answer: A (Mine: D) Explanation: The shopper will prevail because the employee of the contractor hired by the store left the level in the aisle. The general rule that a principal will not be vicariously liable for the acts of its independent contractor's agent is subject to several broad exceptions, including one for duties that are nondelegable because of public policy considerations. One of these duties is the duty of a business to keep its premises safe for customers. Hence, a business would be liable for the negligence of an employee of an independent contractor to the same extent as for the negligence of its own employee. Here, the carpenter was employed by the contractor, which was hired by the store. The carpenter breached the duty owed to customers such as the shopper by leaving the level projecting out into one of the aisles. The shopper was injured as a result, so he will prevail in a suit against the store. (B) is wrong. As part of the duty owed to customers, the store employees have a duty to make reasonable inspections of their premises to discover unsafe conditions (such as if a customer had spilled something slippery in an aisle). ***However, regardless of whether they had a reasonable time to discover the level, the store is liable because it is responsible for the carpenter's conduct. (C) is wrong because, as discussed above, the store is liable under these circumstances for the conduct of its independent contractor's employee. (D) is wrong because the store is liable regardless of the knowledge of its employees.

A shopper at a grocery store slipped and fell when he stepped in some water that had seeped out from a malfunctioning freezer case. The fall caused the shopper to break an ankle, so he filed suit against the store in a jurisdiction applying the traditional rules for landowners and possessors of land. At trial, the shopper presented evidence of the above facts, and testified that the floor around the water appeared dirty. To survive a motion for summary judgment by the store, what additional evidence must the shopper present? response - incorrect A No additional evidence. B He was planning to make a purchase at the store. C The store employees knew that the freezer case was leaking. D His attention was diverted by store displays so that he did not notice the water on the floor.

Answer: A (Mine: D) Explanation: The shopper's lawsuit will survive a motion for summary judgment by the store without any additional evidence. Under the facts here, the shopper was an invitee as to the store because he came onto the premises for a purpose connected with the store's business. The store therefore owed him the duty to warn of nonobvious dangerous conditions and to make reasonable inspections to discover dangerous conditions and make them safe. The shopper's testimony that the floor around the water appeared dirty suggests that the floor had not been swept or mopped for some time. This is enough evidence to allow the jury to decide whether the store employees failed to reasonably inspect or make safe an area in which its invitees would walk, which would breach its duty to the shopper. (B) is incorrect because the shopper need not show that he planned to make a purchase to have the status of an invitee. Even if he came just to return an item or browse the aisles and compare prices, he qualifies as an invitee. (C) is incorrect because the store could be liable even if its employees did not know that water was leaking onto the floor. Because the shopper was an invitee, the store owed a duty to make reasonable inspections to discover unsafe conditions. ***(D) is incorrect because the shopper need not establish his due care here. Even if the shopper was not distracted by displays and should have seen the water had he been watching where he was walking, he can still recover some damages under pure comparative negligence, which allows recovery against a negligent defendant no matter how great plaintiff's negligence is. ***It will be an issue for the jury to determine whether and to what extent the shopper was at fault.

A small cruise ship struck a whale swimming underwater, causing the ship to suddenly lurch sideways. A passenger on the ship who was walking down a corridor lost his balance and bumped his head on the edge of a doorway. Because of a previously existing medical condition that made him susceptible to bleeding on the brain, he suffered a cerebral hemorrhage and permanent mental impairment, despite prompt medical attention on the ship. The passenger brought suit against the cruise ship owner for his damages. At trial, the passenger presented evidence of how he was injured as he walked down the hallway, his previous medical condition, and his medical expenses and other damages. The cruise ship owner presented evidence that the cruise ship was following its approved route and that the whale could not have been detected before impact, and that the bump would not have injured someone in ordinary health. At the close of the evidence, the cruise ship owner moved for a directed verdict. How should the court rule? A Grant the motion, because there is no evidence that the crew operated the ship negligently. B Grant the motion, because the cruise ship owner introduced uncontroverted evidence that a person in normal health would not have been injured by the bump. C Deny the motion, because the jury could find that the cruise ship owner, as a common carrier and innkeeper, breached its high duty of care to the passenger. D Deny the motion, because the fact that the severity of the passenger's injuries was not foreseeable does not cut off the cruise ship owner's liability.

Answer: A. Explanation: The court should grant the cruise ship owner's motion because the passenger has not established a prima facie case of negligence against the cruise ship. To establish a prima facie case for negligence, a plaintiff must show (i) a duty of care, (ii) breach of that duty, (iii) actual and proximate cause, and (iv) damages. As a common carrier and/or an innkeeper, the cruise ship owed its passengers a high duty of care, and therefore would be liable for slight negligence. However, the passenger has offered no evidence to establish that the cruise ship employees breached that duty, and res ipsa loquitur is not applicable here because the collision with the whale swimming underwater is not the type of event that would occur only as a result of negligence. Because the passenger failed to establish breach of duty, the court should grant the cruise ship owner a directed verdict. (B) is incorrect because the cruise ship owner does not need that evidence to prevail. While evidence that a person in normal health would not have been injured by the bump supports the cruise ship's other evidence that it exercised due care, it is not necessary because the passenger has failed to offer evidence that the cruise ship owner breached its duty. On the other hand, if the cruise ship owner had breached its duty of care to its passengers, the fact that a person in normal health would not have been injured by the bump on the head would not be a defense to liability. If a defendant's negligence causes an aggravation of a plaintiff's existing physical illness, the defendant is liable for the damages caused by the aggravation. (C) is incorrect because, as discussed above, the passenger has failed to present evidence that the cruise ship owner breached the high duty of care that it owed to its guests. (D) is incorrect even though it is a true statement of law, as discussed above. The reason the cruise ship owner prevails is because the passenger has failed to establish a prima facie case.

A motorist was driving along a narrow, winding road when his car ran out of gas. Because the road had no shoulders, the motorist pushed his car onto the driveway of a landowner. Finding no one home at the house, the motorist started walking toward a gas station he had passed a mile back. While he was gone, the landowner returned and found the car in his driveway, with two of its wheels partially on his flower garden. Although the landowner had not posted any "no trespassing" signs, he believed he had the right to remove the car from his property. The car was unlocked, so he released the parking brake and pushed the car back onto the edge of the road, and then reset the brake. Before the motorist could return, a truck had sideswiped the car, damaging it. Can the motorist recover against the landowner for the damage to his car? response - correct A Yes, because the landowner had not posted any "no trespassing" signs on his property. B Yes, because the motorist was privileged to leave his car there. C No, because the motorist's car damaged the landowner's property. D No, because the landowner reasonably believed that he had a right to remove the car from his property.

Answer: B Explanation: The motorist can recover against the landowner because the privilege of private necessity applied. A person may interfere with the real or personal property of another when the interference is reasonably and apparently necessary to avoid threatened injury from a natural or other force and the threatened injury is substantially more serious than the invasion that is undertaken to avert it. Here, it was necessary for the motorist to push the car into the landowner's driveway to avoid the threat of other vehicles colliding with it on the narrow road, which would be a substantially more serious harm than any damage to the landowner's property. Hence, the landowner was not entitled to move the car back into the road, and the motorist can recover from the landowner the damages that resulted from doing so. (A) is wrong because the absence of "no trespassing" signs is irrelevant. Even if the landowner had posted "no trespassing" signs and the motorist saw them, the signs do not negate the privilege, which supersedes any right of the landowner to protect his property. (C) is similarly incorrect; the fact that the motorist's car damaged the landowner's property does not extinguish the privilege. The motorist will be required to pay for the damage to the landowner's flower bed because the privilege of private necessity is not absolute, but the landowner was not entitled to move the car off of his land and back into danger. (D) is incorrect. A landowner's reasonable belief that he had a right to defend his property generally is not a defense to the entrant's exercise of a privilege, such as necessity, that supersedes the defense of property right. Here, the landowner's reasonable mistake that he had a right to remove the car is no defense.

A patient troubled by an irritating skin rash consulted a dermatologist for treatment. The dermatologist diagnosed the rash as a genetic condition that had no cure and would ultimately spread and lead to disfigurement. The patient was shocked and distressed by the diagnosis. On the advice of her family, a week later the patient consulted another doctor. That doctor immediately diagnosed the skin rash as a common bacterial infection and prescribed an ointment that cleared up the condition in a few days. Because the doctor was a friend of the family, the patient was not charged for that visit. Can the patient recover from the dermatologist for the emotional distress caused by his erroneous diagnosis? A Yes, but only if the patient's distress caused her some physical injury. B Yes, because the misdiagnosis by the dermatologist caused the patient actual harm. C No, because the patient did not have to pay for the second doctor visit. D No, because the dermatologist's conduct did not create a foreseeable risk of physical injury to the patient.

Answer: B Explanation: The patient's distress is a recoverable element of damages caused by the dermatologist's breach of duty to her. A doctor owes a duty to possess and exercise the degree of knowledge and skill exercised by other doctors in good standing. The dermatologist also owes a duty to exercise the superior knowledge and skill that he possessed in his area of specialty. He breached his duty by misdiagnosing a common skin infection that another doctor was able to diagnose immediately. His failure to properly diagnose the condition was the actual and proximate cause of injury to the patient; but for the misdiagnosis, she would not have had to continue suffering from the rash until the other doctor properly treated it. *The continuation of the rash and any pain and suffering from it are compensable damages that she can recover from the dermatologist. *Also compensable is the emotional distress that she suffered because of the misdiagnosis. While recovery for negligent infliction of emotional distress is not always available in many jurisdictions when there is no other injury caused by the breach, ***these restrictions do not apply when plaintiff is the victim of another tort that causes physical injury. Plaintiff can recover damages for emotional distress that arise from the tortious conduct. Hence, (A) is incorrect. (C) is incorrect because the patient has suffered compensable injury regardless of whether she had to pay for the second doctor visit. **The continuation of the skin rash until she saw the other doctor suffices as the damage element of the prima facie case. (D) is incorrect because, given the patient's physical condition, a failure to make a proper diagnosis did create a foreseeable risk that she would continue to suffer from a painful condition that could otherwise have been alleviated. Thus, the dermatologist's conduct constituted a breach of duty.

Which of the following need NOT be shown by the plaintiff under the attractive nuisance doctrine? A) O was/s/h/k of the dangerous condition B) Kid = lured onto property by the attractive nuisance C) Condition = likely to cause the injury of the kid's inability to appreciate the risk D) Expense of remedying the situation = sight compared w/magnitude of the risk

Answer: B Explanation: The plaintiff does not need to show that the child was lured onto the property by the attractive nuisance. The plaintiff does need to show that (I) the owner was or should have been aware of the dangerous condition, that (II) it was likely to cause injury because of the child's inability to appreciate the risk, and that (III) the expense of eliminating the danger is slight compared with the magnitude of the risk. Under the attractive nuisance doctrine, a landowner has a duty to exercise ordinary care to avoid reasonably foreseeable risk of harm to children caused by artificial conditions on the property. To recover under this doctrine, the plaintiff must show that (i) there is a dangerous condition present on the land of which the owner is or should be aware, (ii) the owner knows or should know that young persons frequent the vicinity of this dangerous condition, (iii) the condition is likely to cause injury, i.e., is dangerous, because of the child's inability to appreciate the risk, and (iv) the expense of remedying the situation is slight compared with the magnitude of the risk.

Which of the following best states who may bring a strict liability action against a defendant engaging in abnormally dangerous activities? A Anyone injured as a result of the dangerous propensity of the activity B Any foreseeable plaintiff injured as a result of the dangerous propensity of the activity C Anyone directly injured by the activity D Any foreseeable plaintiff as long as she was directly injured by the activity

Answer: B. Explanation: A defendant engaging in an abnormally dangerous activity may be liable only to foreseeable plaintiffs injured as a result of the dangerous propensity of the activity. This is the best statement of the scope of the duty owed. In most states, a defendant will be liable only to those persons to whom a reasonable person would have foreseen a risk of harm under the circumstances. In general, strict liability is not imposed for injuries to a plaintiff to whom no reasonable person would have foreseen a danger. The defendant will not be strictly liable to all plaintiffs who were directly injured by the activity. Rather, the harm must result from the kind of danger to be anticipated from the abnormally dangerous activity; i.e., it must flow from the "normally dangerous propensity" of the activity involved. Conversely, a foreseeable plaintiff may recover even if she was not directly injured by the activity as long as the injury was from the dangerous propensity (e.g., injuries caused by fleeing the danger from the activity).

A salesman in a highly visible and competitive field went to the police station to post bond for his son, who had been arrested for possession of a small quantity of narcotics. A photographer for the local newspaper who was at the police station took a picture of the salesman flanked by two bulky police officers. The photo, which looked like the pictures of alleged criminals being taken into custody, ran on a quarter of the front page because it had been a slow news day. The photo was accompanied by a very small caption giving the salesman's name and stating that his son had been arrested for possession of narcotics. The salesman's boss was hypersensitive about the reputation of his company and fired the salesman after he saw the picture in the newspaper. If the salesman brings an invasion of privacy action against the newspaper, what is the most likely basis? A Intrusion upon seclusion B False light publicity C Public disclosure of private facts D Appropriation of plaintiff's picture for commercial purposes

Answer: B Explanation: The salesman's basis for an invasion of privacy action will be that the newspaper published facts about the salesman that placed him in a false light. To establish a prima facie case for invasion of privacy based on publication by defendant of facts placing plaintiff in a false light, the following elements must be proved: (i) publication of facts about plaintiff by defendant placing plaintiff in a false light in the public eye; and (ii) the "false light" must be something that would be highly offensive to a reasonable person under the circumstances. The large picture of the salesman flanked by two bulky police officers could suggest that the salesman committed a crime because it looked like pictures that newspapers often print of alleged criminals being taken into custody.This "false light" would be highly offensive to a reasonable person under the circumstances. (A) is incorrect. This branch of invasion of right to privacy, intrusion upon plaintiff's affairs or seclusion, requires (i) an act of prying or intruding on the affairs or seclusion of plaintiff by defendant; (ii) the intrusion must be something that would be highly offensive to a reasonable person; and (iii) the thing to which there is an intrusion or prying must be "private." Here, the photograph of the salesman was taken at the police station, which is a public place. Hence, the intrusion was not into anything of the salesman's private domain and is not actionable under this branch of invasion of privacy. (C) is similarly incorrect. Public disclosure of private facts requires (i) publication or public disclosure of private information about the plaintiff, and (ii) the matter made public is such that its disclosure would be highly offensive to a reasonable person. Here, the presence of the salesman outside the police station was not a private fact (& the implication was not true; hence, not a fact) (D) is incorrect because appropriation of a plaintiff's picture or name for commercial purposes must be for the promotion or advertisement of a product or service; the fact that the defendant is using the picture in a newspaper that it is selling is not sufficient.

A swimmer went to a privately owned lake resort whose owner charged a fee for admission. The beach had a roped-in swimming area and large signs directing swimmers not to swim anywhere but within the ropes. The lifeguards regularly enforced this rule. The resort also rented canoes and rowboats to its patrons, who could take them anywhere on the lake. The swimmer and two of his friends had rented a canoe and started to paddle out toward the other side of the lake when the swimmer saw a volleyball game starting on the beach that he wanted to join. He left his friends in the canoe and started swimming to shore. He was only a few yards outside of the roped-in swimming area when he started, but he angled away from the swimming area toward the area of the beach where the volleyball net was set up. Although the lifeguard on duty saw him, she did not warn him to return to the swimming area. When the depth of the water was about four feet, he put his foot down and was severely cut by the jagged edge of a rusted metal stake protruding a few inches out of the bottom of the lake. The swimmer had not seen the stake even though the water was clear and it was visible if he had looked down. If the swimmer sues the resort for his injury in a jurisdiction that applies the traditional rules for landowners and possessors of land, is he likely to recover? A No, because the stake could have been seen by the swimmer. B No, because he was swimming outside of the roped-in area. C Yes, because the lifeguard on duty saw him and did not warn him to return to the swimming area. D Yes, because he is a public invitee of the resort.

Answer: B Explanation: The swimmer cannot recover from the resort because he did not have invitee status when he was injured. In jurisdictions following the traditional rules for landowners and possessors of land, the nature of a duty of an owner or occupier of land to those on the premises depends on the legal status of the plaintiff in regard to the property, i.e., whether the plaintiff is a trespasser, licensee, or invitee. An invitee is a person who enters onto the premises in response to an express or implied invitation of the landowner. Those who enter as members of the public for a purpose for which the land is held open to the public and those who enter for a purpose connected with the business or other interests of the landowner are considered invitees. However, a person will lose his status as an invitee if he exceeds the scope of the invitation—if he goes onto a portion of the property where his invitation cannot reasonably be said to extend. Here, the swimmer was an invitee of the resort in the areas to which it allowed its patrons to go. However, the resort clearly identified the boundaries of the area held open to swimmers, and the swimmer could not reasonably have believed that he was invited to swim in the area where he was injured. Because the swimmer was at most a licensee when he was injured, the resort did not owe him a duty to make reasonable inspections of that area to discover dangerous conditions and make them safe. At most, the resort had a duty only to warn the swimmer of known dangerous conditions that create an unreasonable risk of harm to him and that he is unlikely to discover, and nothing in the facts indicates that any employees of the resort knew of the stake under the water. The swimmer therefore cannot recover against the resort. (A) is not as good a choice as (B). While a landowner is not liable for a dangerous condition that is obvious to the entrant on the land, the fact that the stake was visible does not establish that it was obvious, given that the swimmer was looking forward rather than down. Whether a danger is obvious is determined by all of the surrounding circumstances, not just whether the danger is visible. The better reason why the swimmer cannot recover is because he was no longer an invitee. (C) is incorrect because the lifeguard's failure to direct the swimmer to the swimming area would not constitute an invitation to swim in the restricted area; at most, it would establish only that the swimmer was a licensee rather than a trespasser when he swam in that area. A licensee is one who enters onto land with the possessor's permission, express or implied, for his own purpose or business rather than for the possessor's benefit. The lifeguard's conduct may have constituted implied permission for the swimmer to exit the lake in a nonswimming area for his own benefit, but it does not establish that he reasonably believed that he was invited to swim in that area. (D) is incorrect because the swimmer lost his status as an invitee when he exceeded the scope of his invitation by swimming in an area where swimming was not permitted.

In contrast to products liability cases based on negligence, those based on strict liability do not: A Require an injured bystander to be foreseeable. B Require that suppliers have an opportunity to inspect. C Prohibit recovery of solely economic losses. D Impose liability when an intermediary negligently failed to discover the defect.

Answer: B Explanation: Unlike with products liability cases based on negligence, those based on strict liability do not require that suppliers have an opportunity to inspect. Thus, for a case based on the sale of a defective product, a retailer in a strict liability action may be liable for a manufacturing or design defect simply for being a commercial supplier of that defective product, even if it had no opportunity to inspect the manufacturer's product before selling it. In a negligence action, the supplier's negligence must be proved. Products liability cases based on negligence and those based on strict liability BOTH require that an injured bystander be foreseeable. While privity is not required in these cases, and bystanders are protected and may bring a claim under either theory, they must be foreseeable plaintiffs. Liability under these theories applies only to foreseeable plaintiffs. Products liability cases based on negligence and those based on strict liability both prohibit recovery of solely economic losses. The types of damages recoverable under both theories are the same: personal injury and property damages. Economic loss cannot be the sole damage claim. As under claims based on negligence, those based on strict liability will impose liability even though an intermediary negligently failed to discover the defect. The same concepts of proximate cause govern negligence and strict liability actions. The negligent failure of an intermediary to discover a defect is not a superseding cause and does not cut off the supplier's strict liability. However, if the intermediary's conduct becomes something more than ordinary foreseeable negligence, then it does become a superseding cause.

The driver of a tanker truck was transporting radioactive waste from a nuclear power plant to a permanent storage facility in a remote western region of the United States. After driving all night, the driver fell asleep at the wheel and the truck crossed over the center line, off the road, and onto a homeowner's property, coming to rest after crashing into several glass cases containing the homeowner's collection of poisonous snakes, the keeping of which was permitted by local ordinance. When the driver exited the truck, he was bitten on the leg by one of the poisonous snakes and became seriously ill. The driver brought an action against the homeowner for his injuries. The parties stipulated to the above facts, and that the driver violated a state statute by driving off of the road. Both parties moved for judgment as a matter of law on the liability issue. How should the court rule? response - incorrect A Grant the driver's motion and deny the homeowner's motion, because the homeowner is strictly liable for the injury caused by the snake. B Deny the driver's motion and grant the homeowner's motion, because the driver was a trespasser on the homeowner's property. C Deny the driver's motion and grant the homeowner's motion, because the driver's violation of the state statute establishes contributory negligence as a matter of law. D Deny both parties' motions, because both parties were engaged in an activity for which strict liability is imposed.

Answer: B (Mine: A) Explanation: The court should grant the homeowner's motion for judgment as a matter of law because the driver has not established a prima facie case against the homeowner. An owner of wild (dangerous) animals is strictly liable for injuries caused by those animals as long as the person injured did nothing, voluntarily or consciously, to bring about the injury. However, strict liability generally is ****not imposed in favor of undiscovered trespassers against landowners in the absence of negligence, such as when the landowner knows that the trespassers are on the land and fails to warn them of the animal. Here, despite the fact that the driver did not intend to enter the homeowner's land (and thus would not be liable for the intentional tort of trespass), his status on the homeowner's land is that of a trespasser rather than a licensee or invitee. The driver has presented no evidence of negligence on the homeowner's part and therefore has not established a prima facie case against the homeowner. (A) is wrong because, as discussed above, the homeowner is not strictly liable to the driver because the driver was a trespasser. (C) is incorrect because the driver will not prevail regardless of whether he was contributorily negligent, because he cannot establish a prima facie case against the homeowner in either negligence or strict liability. (D) is incorrect for several reasons: While the driver's transport of radioactive waste may have been an abnormally dangerous activity, that danger had nothing to do with the accident that occurred. Furthermore, the fact that the driver may have been engaged in an abnormally dangerous activity would not prevent him from recovering damages from another tortfeasor if he established the requisite prima facie case. Finally, the fact that the parties were engaged in activities potentially creating strict liability has nothing to do with whether issues of fact regarding liability still exist that would require denying both motions and going to trial.

A doorman negligently locked a door that an office worker was intending to use to exit an office building, so the worker was forced to use a different exit. As she stepped onto the sidewalk outside the building, a car careened out of control on the street and jumped the curb. The car struck and injured the worker and then drove off. The driver was not found. The worker brought suit against the doorman, seeking damages for her injuries. At trial, the parties stipulated that the doorman was negligent in locking the door and that the worker suffered injuries when she was struck by the car. The worker also established that if she had exited from the door she was intending to, she would not have been struck by the car. At the end of the worker's case, the doorman moved for a directed verdict in his favor. How should the judge rule? A Grant the motion, because the driver of the car was the actual cause of the worker's injuries. B Grant the motion, because the car was an unforeseeable intervening force. C Deny the motion, because the jury could find that but for the doorman's negligence, the worker would not have been injured. D Deny the motion, because the jury could find that the doorman's negligence was a foreseeable concurring cause of the worker's injury.

Answer: B (Mine: D) Explanation: The court should grant the motion because the evidence establishes that the car was a superseding force that cut off the doorman's liability for his negligence under proximate cause principles. The general rule of proximate cause is that the defendant is liable for all harmful results that are the normal incidents of, and within the increased risk caused by, his acts. An indirect cause case is one where the facts indicate that a force came into motion after the time of defendant's negligent act and combined with the negligent act to cause injury to the plaintiff. Whether an intervening force will cut off the defendant's liability for the plaintiff's injury and be deemed superseding is determined by foreseeability. Here, nothing in the facts suggests that a car jumping the curb was a foreseeable consequence of the doorman's negligently locking the door. Hence, the judge should grant the motion because the worker has failed to establish the proximate cause element of his prima facie case. (A) is wrong because the doorman was also an actual cause of the worker's injuries—but for the doorman's negligence, the worker would not have been on the sidewalk where the car jumped the curb. (C) is wrong because it establishes only actual cause. A directed verdict is appropriate because no evidence establishes the proximate cause element of the worker's case. ***(D) is wrong because the facts do not establish foreseeability. While the doorman's negligence was a concurring actual cause of the worker's injury, it was not a proximate cause because the injury that occurred was unforeseeable.

To show breach of duty, a plaintiff may rely on evidence of any of the following except: A Res ipsa loquitur B Violation of a statute C A "Good Samaritan" law D Custom or usage

Answer: C Explanation: A "Good Samaritan" law has nothing to do with breach of duty. It refers to a statute exempting licensed doctors, nurses, etc., who voluntarily and gratuitously render emergency treatment, from liability for ordinary negligence. Hence, it would not be used to establish breach of duty. To prove breach of duty, it must be shown what in fact happened, and (based on these facts) that the defendant acted unreasonably. Proof of what happened may be established by either direct or circumstantial evidence. Other matters may also be offered into evidence to establish the standard by which defendant's conduct is to be measured, such as: 1. Custom or usage; 2. Violation of an applicable statute; and 3. The circumstantial evidence doctrine of res ipsa loquitur.

When a statutory standard of care replaces the common law duty of care in a negligence case, the defendant's lack of compliance with the statute will be excused if: A The defendant otherwise exercised due care B Violation of the statute causes more danger than compliance C Compliance with the statute is beyond the defendant's control D Violation of the statute would result in a civil infraction instead of a criminal one

Answer: C Explanation: A violation of an applicable statute may be excused if: (i) compliance with the statute would cause more danger than a violation (e.g., a defendant drives onto the wrong side of the road to avoid hitting children who dart into his path), or (ii) compliance with the statute would be beyond the defendant's control (e.g., a blind pedestrian crosses against a light). Violation of the statute will not be excused if violation of the statute causes more danger than compliance; this misstates the rule stated above. It is not correct to state that violation of the statute will be excused if violation of the statute would result in a civil infraction instead of a criminal one; ***rather, the statute itself supplies the remedy rather than damages through a common law negligence action. The defendant will not be excused for violating the statute. Violation of the statute will not be excused if the defendant otherwise exercised due care. When a statute is applicable, the ordinary standard of due care is replaced by the standard of care established by the statute.

In a negligence action, the plaintiff cannot recover: A Unforeseeable damages B Noneconomic damages C Presumed damages D Damages for lost future earning capacity

Answer: C Explanation: In a negligence action, the plaintiff cannot recover presumed damages. Damage is an essential element of a plaintiff's prima facie case for negligence. This means actual harm or injury. Unlike for some intentional torts, damage will not be presumed in negligence. A plaintiff is entitled to all damages that he can prove, even if the extent of the damages was unforeseeable [but not if P = unforeseeable] Permissible damages includes economic damages, such as medical expenses and lost earnings, and noneconomic damages, such as pain and suffering. The plaintiff is also entitled to damages for lost future earning capacity, discounted to present value to avoid an excess award; i.e., the plaintiff receives an amount that, if securely invested, would produce the income that the jury wishes him to have.

In a products liability case based in strict liablity, a plaintiff may recover: A Only damages for economic losses B Only personal injury damages C Personal injury damages and property damages D Personal injury damages, property damages, and damages for economic losses

Answer: C Explanation: In a products liability case based on strict liability, a plaintiff may recover both personal injury damages and property damages for the supplying of a defective product. If the plaintiff's complaint is only that the product does not work as well as expected or requires repairs (i.e., no personal injury or property damages), most courts do not permit recovery of damages for economic losses under either a strict liability or a negligence theory; the plaintiff must bring an action for breach of warranty.

The right of contribution among tortfeasors: A Imposes contribution based on equal shares of the overall liability. B Provides for apportionment of damages in the absence of joint and several liability. C Does not apply against a tortfeasor who is immune from liability. D Applies to intentional torts.

Answer: C Explanation: The right of contribution among tortfeasors is a device whereby responsibility is apportioned among those who are at fault. However, it does not apply against a tortfeasor who is immune from liability. If the contributing tortfeasor has a defense that would bar liability, such as intra-family tort immunity, she is not liable for contribution. In most states, contribution is based on relative fault of the various tortfeasors rather than on equal shares of the overall liability. Contribution does not provide for apportionment of damages in the absence of joint and several liability; rather, it can only operate in response to joint and several liability, because it allows any tortfeasor required to pay more than his share of damages under joint and several liability rules to have a claim against the other jointly liable parties for the excess. Contribution does not apply to intentional torts in most states.

Which of the following describes only dependent intervening forces in a proximate cause analysis? A Efforts to protect person or property and acts of God. B Subsequent medical malpractice and criminal acts of third persons. C A subsequent disease and negligence of rescuers. D A subsequent accident and an intentional tort of a third person.

Answer: C (LL-ONLY: a) acts of God, b)most crime c)most intentional torts; E--D's Neg creates an Unforeseeable risk that unforeseeable injury types would occur) Explanation: Dependent intervening forces are normal responses or reactions to the situation created by the defendant's negligent act. Dependent intervening forces are almost always foreseeable. A subsequent disease is a common dependent intervening force. The original tortfeasor is usually liable for diseases caused in part by the weakened condition in which the defendant has placed the plaintiff by negligently injuring her. Also, negligence of rescuers is a common dependent intervening force. Generally rescuers are viewed as foreseeable intervening forces, so the original tortfeasor usually is liable for their negligence. Efforts to protect person or property are common dependent intervening forces. A defendant is usually liable for negligent efforts on the part of persons to protect the life or property of themselves or third persons endangered by the defendant's negligence. Subsequent medical malpractice is also a common dependent intervening force. The defendant is usually liable for the aggravation of the plaintiff's condition caused by the malpractice of the treating physician. A subsequent accident may also be a dependent intervening force if the original injury was a substantial factor in causing the second accident. However, acts of God and intentional torts and criminal acts of third persons are independent intervening forces. Independent intervening forces operate on the situation created by the defendant's negligence, but they are independent actions rather than natural responses or reactions to the situation. (Note that the defendant may or may not be liable for independent intervening forces. It depends on whether they are foreseeable.) Thus, the other choices are incorrect because they list both dependent and independent intervening forces.

A bolt of lightning struck a tree, causing it to fall on a farmer's fence which enclosed a pasture containing the farmer's large bull. The bull escaped through the broken fence and entered the neighbor's property. It gored a hiker who was crossing the neighbor's property without permission. In the hiker's action against the farmer based on strict liability, is the hiker likely to prevail? response - incorrect A Yes, because the bull caused harm while trespassing on another's property. B Yes, because bulls have known dangerous propensities. C No, because a bull is a domestic animal. D No, because the hiker was a trespasser.

Answer: C (Mine: A) Explanation: The hiker will not prevail because strict liability does not apply to a bull, which is a domestic animal. The owner of a domestic animal, including a farm animal, is not strictly liable for injuries it causes, as long as the owner has no knowledge that the animal has abnormally dangerous propensities (i.e., propensities more dangerous than normal for that species). A bull is a domestic animal, and nothing in the facts suggests that the bull ***was more dangerous than normal for that type of animal. Hence, strict liability will not apply. (A) is incorrect because the rule for trespassing animals does not apply. The owner of a trespassing animal is strictly liable for harm done by the trespass as long as it was ****reasonably foreseeable. Here, the bolt of lightning caused the fence to break and allowed the bull to escape. This unforeseeable intervening force was the cause of the trespass; hence, the strict liability rule for trespassing animals does not apply here. (B) is incorrect because, as discussed above, strict liability does not apply for domestic animals with normal dangerous propensities. ***Only domestic animals with propensities more dangerous than normal for the species may subject the owner to strict liability. (D) is incorrect because the hiker's status as a trespasser on the neighbor's land is irrelevant as to the farmer's liability. If the hiker were a trespasser on the farmer's land, strict liability would not apply even if the bull were abnormally dangerous, but the farmer's liability is not affected by the hiker's status as to the neighbor. Note that if strict liability applied for harm from an animal trespassing on a neighbor's property, the hiker's status as a trespasser might be relevant because ****strict liability [based on a trespassing animal] applies only to injured persons who were rightfully on the property. However, as discussed above in (A), that liability is inapplicable here because the bull's trespass was unforeseeable.

Which of the following statements regarding proximate cause is true? response - incorrect A In indirect cause cases, another force comes into play before the defendant's negligent act and combines with it to cause the injury. B A defendant may have proximately caused the plaintiff's injury even though she did not actually cause it. C In direct cause cases, the unusual manner in which the injury occurred is not relevant. D Negligence of rescuers generally is not foreseeable.

Answer: C (Mine: A; not "another force" but an "affirmative intervening force"; o/wise not wrong) Explanation: A direct cause case is one where the facts present an uninterrupted chain of events from the time of the defendant's negligent act to the time of plaintiff's injury. If a particular harmful result was at all foreseeable from the defendant's negligent conduct, the unusual timing of cause and effect or the unusual manner in which the injury occurred is not relevant to the defendant's liability. Negligence of rescuers is generally foreseeable because a rescue attempt is a common intervening force that is a normal response or reaction to the situation created by the defendant's negligent act (i.e., "danger invites rescue"). It is not true that a defendant may have proximately caused the plaintiff's injury even though she did not actually cause it. In fact, the opposite is true: A defendant may have actually caused the plaintiff's injury but not have proximately caused the injury, because proximate cause is a limitation on liability. In indirect cause cases, another force comes into play AFTER the defendant's negligent act and combines with it to cause the injury.

Which of the following situations involves a common intervening force that courts almost always find foreseeable? response - incorrect A A roofer negligently leaves a hammer on the plaintiff's roof, and a strong wind blows the hammer off the roof, where it strikes the plaintiff B A parking lot attendant negligently leaves the keys to the plaintiff's car inside it with the doors unlocked, and a thief steals the car C A defendant negligently causes a plaintiff to break her leg, and while walking on her crutches, the plaintiff loses her balance and injures herself D A defendant negligently blocks a sidewalk, forcing the plaintiff to walk in the roadway, where he is struck by a negligently driven car

Answer: C (Mine: D) Explanation: If the plaintiff loses her balance while on crutches and injures herself, this involves an intervening force that is a normal response or reaction to the situation created by the defendant's act. This is a subsequent accident situation, where the plaintiff suffers a subsequent injury following her original injury, and the original injury was a substantial factor in causing the second accident. This is a type of intervening force that courts almost always find foreseeable. The other situations involve intervening forces that operate on a situation created by a defendant's negligence but are independent actions, rather than natural responses or reactions to the situation. An independent negligent act of a third person, a criminal act of a third person, and an act of God are all independent intervening forces. While these may be found to be foreseeable if the defendant's negligence created a significant risk that these forces would cause harm to the plaintiff, other types of intervening forces are deemed foreseeable because they are normal responses or reactions to the situation created by the defendant's act.

A fire broke out in a home that had been recently remodeled, destroying the house and injuring the homeowner. An investigation by the fire marshal established that the fire started from a short in some wiring behind a wall. A small section of wiring that ran to an outlet through a narrow gap between a furnace chimney and a hot water pipe had had part of its outer sheath cut off. The homeowner filed suit against the electrical company that did the rough wiring. The parties stipulated for trial that the company had installed the wiring in compliance with the blueprints, and that the wiring had been inspected and approved by the building inspector before the chimney and the water pipe had been installed and the walls put up, all by different contractors. At trial, the homeowner introduced the report of the fire marshal establishing how the fire started, and evidence of his medical expenses and other damages. At the end of the homeowner's case, the electrical company's attorney rested her case and moved for a directed verdict. The homeowner's attorney also moved for a directed verdict. How should the court rule on the directed verdict motions? A Deny the electrical company's motion and grant the homeowner's motion for a directed verdict, because a short in the wiring caused the homeowner's injuries. B Deny the electrical company's motion and grant the homeowner's motion for a directed verdict, because the company failed to rebut the presumption of negligence that the homeowner has established. C Deny the homeowner's motion and grant the electrical company's motion for a directed verdict, because the wire could have been damaged by another contractor. D Deny both directed verdict motions, because the homeowner has presented enough evidence to submit the case to the jury.

Answer: C (Mine: D) Explanation: The court should grant the electrical company's motion for a directed verdict because the homeowner has not established a prima facie case of negligence on the company's part. The homeowner has established that the electrical company owed a duty to him and that he has suffered harm from the fire caused by the short in the wiring. However, he has not established that the company breached any duty to him. While breach of duty is ordinarily a question for the trier of fact, a plaintiff's failure to offer any evidence on that element of the prima facie case will permit a directed verdict for the defendant. Under certain circumstances, the fact that a particular injury occurred may itself establish or tend to establish a breach of duty owed, permitting the trier of fact to infer the defendant's liability. This is the doctrine of res ipsa loquitur ("the thing speaks for itself"). However, for the doctrine to apply, the plaintiff must show that (i) the accident causing his injury is the type that would not normally occur unless someone was negligent; (ii) the negligence was attributable to the defendant; and (iii) the injury was not attributable to the plaintiff. The second requirement can often be satisfied by showing that the instrumentality causing the injury was in the exclusive control of the defendant. Here, however, the wiring was exposed to work done by other contractors in installing a chimney and a hot water pipe nearby and putting up the walls, and the homeowner has offered no evidence that the cut in the outer sheath of the wiring was present when the electrical company finished its work. Instead, the fact that the wiring had been approved by the building inspector suggests that the wiring was intact when the electrical company finished. Given these facts, the homeowner has not presented evidence that the negligence was attributable to the defendant. Since res ipsa loquitur does not apply and no other evidence of breach of duty was established, the electrical company's motion for a directed verdict should be granted.

For purposes of private nuisance, the interference with the plaintiff's use of the land is unreasonable only if: A The interference is offensive or annoying to an average person in the community. B The remedy of damages is unavailable or inadequate. C The nuisance is a "nuisance per se." D The severity of the plaintiff's inflicted injury outweighs the utility of the defendant's conduct.

Answer: D Explanation: The interference with the plaintiff's use of the land will be considered unreasonable under nuisance law when the severity of the plaintiff's inflicted injury outweighs the utility of the defendant's conduct. In balancing these respective interests, courts take into account that every person is entitled to use his own land in a reasonable way, considering the neighborhood, land values, and existence of any alternative courses of conduct open to the defendant. Whether the interference is offensive or annoying to an average person in the community is the test for whether the interference is substantial, which is a separate requirement for establishing a nuisance. Whether the remedy of damages is unavailable or inadequate determines only whether the plaintiff may be able to obtain an injunction remedy. A nuisance is sometimes called a "nuisance per se" when it is based on strict liability (e.g., a nuisance arising from an abnormally dangerous activity).

Which of the following may prevent establishing causation against a manufacturer in a strict products liability action? A The retailer's labeling of the product as its own B The destruction of the product because of its dangerous defect C The failure of a retailer to take action after discovering a dangerous defect D The negligent failure of a retailer to discover a dangerous defect

Answer: C (mine: A) Explanation: The failure of a retailer to take action after discovering a dangerous defect may prevent establishing causation against a manufacturer in a strict products liability action. The same concepts of proximate cause that govern negligence and strict liability actions are applicable to strict liability actions for defective products. As with products liability cases based on negligence, the negligent failure of a retailer to discover a dangerous defect does not cut off the supplier's strict liability. On the other hand, when the intermediary's conduct becomes something more than ordinary foreseeable negligence, it becomes a superseding cause. Hence, the conduct of a retailer who discovered a dangerous defect and then took no action (such as ***alerting the manufacturer, ***warning the consumer, or ***removing the product from sale) constitutes more than ordinary foreseeable negligence and may cut off the manufacturer's liability. The destruction of the product because of its dangerous defect does not prevent establishing actual cause. If the product has been destroyed, the plaintiff may rely on an inference that this type of product failure ordinarily would occur only as a result of a product defect. The retailer's labeling of the product as its own will not affect the liability of the manufacturer.

A man boarded a plane with his pet rattlesnake hidden in his carry-on bag. The man was not aware that the carry-on bag, which he put under the seat in front of him, had a defective zipper. The snake, which had no teeth or venom and was harmless, escaped from the bag and started slithering down the aisle while the man was using the lavatory. A woman who had just gotten up from her seat saw the snake heading towards her and tried to run in the other direction. She tripped over someone's foot and broke her ankle. If the woman sues the man on a theory of strict liability for her broken ankle, will she prevail? A No, because the snake was in fact a nondangerous animal. B No, because the injury she suffered was not caused by the dangerous propensity of a snake. C Yes, because the snake is a wild animal. D Yes, because it is not a common activity to bring snakes on a plane.

Answer: C (mine: B) Explanation: (C) The woman will prevail because the rattlesnake is classified as a wild animal. An owner of a wild (i.e., nondomestic) animal, even one kept as a pet, will be strictly liable for the damage caused by the animal. A rattlesnake, even a harmless one, will be classified as a wild animal. Therefore (C) is correct and (A) is wrong. (B) is wrong because the injury the woman suffered was within the "normal dangerous propensity" of the animal. Strict liability for wild animals includes liability for the harm that results when a person is attempting to flee from what is perceived to be a dangerous animal. (D) is wrong. The fact that the activity was uncommon in the locale would have some relevance if the lawsuit were based on a theory of strict liability for an abnormally dangerous activity. ***It has nothing to do with strict liability for damage caused by animals.

To assert the defense of property, a defendant using force against another may not: A) use force that may injure the other B) Make a mistake about the right to use force C) Use force w/o a request to desist D) Use force against one w/a privilege to enter the property

Answer: D Explanation: A defendant cannot assert the defense of property if she uses force against one with a privilege to enter the property. Whenever an actor has a privilege to enter upon the land of another because of necessity, right of reentry, right to enter upon another's land to recapture chattels, etc., that privilege supersedes the privilege of the land possessor to defend her property. It is not true that the defendant may not use force that may injure the entrant. The force used must be reasonable and not likely to cause death or serious bodily injury. It is also incorrect to state that the defendant may not use force without a request to desist. A request to desist must usually precede the use of force, but if the circumstances make it clear that the request would be futile or dangerous, then a request to desist is not required. It is also incorrect to state that the defendant may not make a mistake about the right to use force. A reasonable mistake is allowed as to the property owner's right to use force in defense of property, where the mistake involves whether an intrusion has occurred or whether a request to desist is required.

A trainer of homing pigeons brought several of them to a park that he often used for training. He had trained this group of pigeons carefully and was confident that they would readily find their way home. When they were released, one of the pigeons inexplicably turned in the opposite direction from home. Several blocks away at the other end of the park, it collided with a radio-controlled model airplane that its owner had just purchased and was trying out for the first time. The collision sent the airplane out of control; it dipped low across a highway and was struck and run over by a truck. The airplane owner sued the pigeon trainer for the destruction of his airplane. The parties stipulated to the above facts and the airplane owner presented evidence of his damages. The trainer then moved for a directed verdict. Should it be granted? A No, because the trainer's pigeon caused the destruction of the airplane. B No, because the jury could find negligence on the trainer's part under the doctrine of res ipsa loquitur. C Yes, because the truck, rather than the pigeon, was the direct cause of the airplane's destruction. D Yes, because the trainer took reasonable care in training his pigeons.

Answer: D Explanation: The court should grant a directed verdict for the trainer because the airplane owner has not shown that the trainer breached any duty that he owed to him. A prima facie case of negligence requires plaintiff to show the following elements: (i) the existence of a duty on the part of the defendant to conform to a specific standard of conduct for the protection of the plaintiff against unreasonable risk of injury, (ii) breach of that duty by the defendant, (iii) that the breach of duty was the actual and proximate cause of plaintiff's injury, and (iv) damage to plaintiff's person or property. Here, it is doubtful that the trainer's releasing his pigeons created any duty to other users of the park. To the extent that it did, the fact that he had taken great care to train them to return directly to their roosts indicates that he did not breach his duty to the airplane owner. Because the airplane owner has offered no other evidence of negligence, nor any reason to impose strict liability on the trainer (as discussed below), the trainer's motion for a directed verdict should be granted. (A) is incorrect because that choice indicates the imposition of a strict liability standard on the trainer. The owner of a domestic or inherently nondangerous animal is not strictly liable for the injuries it causes. The conduct of the trainer's homing pigeon would not make the trainer liable in the absence of some negligence on his part. (B) is incorrect because the doctrine of res ipsa loquitur applies only to situations where the fact that a particular injury occurred itself establishes that defendant breached a duty. If the doctrine is applicable, no directed verdict may be given for defendant because plaintiff has established a prima facie case. However, the accident must be the type that would not normally occur unless someone was negligent. The collision between the trainer's homing pigeon and the model airplane is not that type of accident; by itself, it provides no suggestion that anyone was negligent. (C) is incorrect because the truck is not a superseding force that breaks "the causal connection" between the action of the trainer's pigeon and the airplane's destruction. In indirect cause cases, where a force came into motion after defendant's act and combined with it to cause injury to plaintiff, defendant will still be potentially liable for foreseeable intervening forces that are within the increased risk caused by his acts. Even if the intervening force is independent (i.e., not a natural response or reaction to the situation), it will be foreseeable where defendant's negligence increased the risk that the independent force would cause harm. Hence, if the trainer were negligent in releasing his pigeon, the fact that the destruction of the airplane was directly caused by the truck would not relieve the trainer from liability, because the initial collision with the pigeon caused the airplane to go out of control and created a substantial risk that it would be damaged by an intervening force.

A state statute required that any freight train operating within the city limits be able to stop within 200 yards of applying its brakes. No fixed speed limit was established or particular type of braking mechanism required, but through either lowered speed or braking power, the 200‑yard limit was required of all trains. Another statute prohibited vehicles from being within the railroad crossing when the lights on the warning signs are flashing or when the gates are lowered. One day, as a freight train was entering the city limits, the engineer saw a car stalled at a street crossing ahead. He immediately applied full braking power, but was unable to stop the train before it had hit and demolished the car. The driver of the car had gotten clear before the impact, but brought suit against the freight line for property damage to the $25,000 car. At trial, the parties stipulated that the car was stalled within the crossing while the warning lights were flashing. Evidence at trial established that the distance from the point at which the engineer applied the train's brakes to the point of impact was 150 yards, and from the braking point to the point at which the train finally stopped was 225 yards. No other evidence of negligence was presented by the driver. At the end of the driver's case, the freight line moved for a directed verdict. Should the court grant the motion? A No, because the freight line was negligent per se. B No, because the freight line was strictly liable for its violation of the braking statute. C Yes, because the driver's car was on the freight line's tracks in violation of the crossing statute. D Yes, because the freight line's violation of the braking statute was not the cause in fact of the accident.

Answer: D Explanation: The court should grant the motion because the driver did not establish the cause‑in‑fact element of his prima facie case against the freight line. The primary test for cause in fact (actual cause) is the "but for" test: An act is the cause in fact of an injury when the injury would not have occurred but for the act. Even though the freight line had a duty created by the statute to be able to stop its train within 200 yards of first braking, and breached that duty (establishing the first two elements of the driver's prima facie case), it must still be shown that the collision would not have occurred in the absence of the breach. **Because the car was only 150 yards from the point of braking, even a train in compliance with the statute would have struck it. Since no other evidence of negligence has been presented, the motion should be granted. (A) is incorrect because establishing the freight line's "negligence per se" through violation of the statute only establishes a conclusive presumption of duty and breach of duty; the plaintiff must still prove causation. (B) is incorrect because generally violation of a statute does not create strict liability; even if it did in this case, the plaintiff would still have to prove causation as part of the prima facie case for strict liability. (C) is not correct because the court will not reach the issue of the plaintiff's contributory negligence in this case because the prima facie case for the defendant's negligence has not been established. Furthermore, establishing the plaintiff's contributory negligence by violation of a statute uses the same rules that govern whether a statute can establish the defendant's negligence. Hence, the driver's violation of the crossing statute may be excused if the trier of fact determines that compliance was beyond his control because his car stalled.

To prove breach of duty in a products liability action based on negligence, the plaintiff must show: A Res ipsa loquitur. B The conduct involved was below the level of care generally exercised by the defendant. C The product was dangerous because it departed from its intended design. D The defendant supplied a defective product.

Answer: D MDefect = Not like the others DDefect = Not as safe as it economically feasibly could be NegDefect = conduct was below level of care generally exercised by reasonable suppliers Explanation: To prove breach of duty in a products liability action, the plaintiff must show (i) negligent conduct by the defendant leading to (ii) the supplying of a defective product by the defendant & (iii) opportunity to inspect by the D supplier. Negligent conduct is demonstrated by showing that the defendant's conduct fell below the standard of care expected of a reasonable person under like circumstances, not the level of care generally exercised by the defendant. To show negligence in a manufacturing defect case, the plaintiff may invoke res ipsa loquitur, but it is not required that the plaintiff prove res ipsa loquitur in establishing breach of duty. A plaintiff may show that a product was dangerous because it departed from its intended design to establish a manufacturing defect, but may instead show that the design itself is deficient (to establish a design defect).

On the way home from a nightclub, a passenger began yelling at the designated driver claiming that he was not taking the best route back to her house. The driver disagreed and contended that his route was the quickest. The passenger impulsively grabbed the steering wheel, causing the car to swerve and strike a pedestrian, injuring him. At trial, the pedestrian established that the driver's license had expired the day before the accident. The driver's traffic record qualified him for an automatic renewal of his driver's license, but he had forgotten to submit it in time. A statute in the jurisdiction makes it an offense to drive a vehicle on any public road in the state without a valid driver's license. Will the pedestrian prevail? A Yes, because the driver violated a statute by driving without a valid license. B Yes, because the driver failed to control his passenger. C No, because the driver did not start the argument. D No, because the passenger's action was the proximate cause of the injury.

Answer: D (Mne: B) Explanation: The pedestrian will not prevail because the passenger's grabbing of the wheel is the negligent conduct that caused the pedestrian's injuries. To establish a prima facie case for negligence, the following elements must be proved: (i) the existence of a duty on the part of the defendant to conform to a specific standard of conduct for the protection of the plaintiff against an unreasonable risk of injury; (ii) breach of that duty by the defendant; (iii) the breach of the duty by defendant was the actual and proximate cause of plaintiff's injury; and (iv) damage to plaintiff's person or property. Here, the driver's actions were an actual cause of the pedestrian's injury because, but for the driver's driving and the passenger's grabbing the steering wheel, the injury to the pedestrian would not have happened. However, the driver's actions were not a proximate cause of the injury because the passenger's grabbing of the steering wheel was a superseding intervening force. A superseding force is one that serves to break the causal connection between the initial wrongful act and the ultimate injury, and itself becomes a direct immediate cause of such injury. Thus, the first actor would be relieved of liability from the consequences of his antecedent conduct. The passenger's conduct in ***SUDDENLY grabbing the steering wheel was an unforeseeable intervening force creating an unforeseeable harmful result, and thus constituted a superseding force. Consequently, the driver would be relieved of any negligence liability since the passenger's actions were the proximate cause of the accident. (A) is incorrect. A clearly stated specific duty imposed by a statute may replace the more general common law duty of due care when (i) the plaintiff is within the class to be protected by the statute, and (ii) the statute was designed to prevent the type of harm suffered. The statute probably does not apply here because it is intended to keep unsafe drivers off the streets, and ***there is no indication that the driver is an unsafe driver, or ***that any driver could have prevented the injury when the passenger grabbed the steering wheel. Even if the statutory standard were applicable, a violation means only that plaintiff will have established a conclusive presumption of duty and breach of duty. It does not, however, establish causation or damages. Here, the fact that the driver does not have a valid license is not the proximate cause of the pedestrian's injury, as discussed above. (B) is incorrect because the driver had no way of knowing that the passenger would grab the steering wheel. The driver's conduct will be measured against that of the ordinary, prudent, reasonable person who drives a vehicle. An ordinary, prudent, reasonable person would not have foreseen that one of his passengers would impulsively grab the steering wheel, and therefore there are no special safety precautions that the driver should have taken as part of his duty of care toward pedestrians. (C) is incorrect because, even if the driver had started the argument, this would not justify the passenger's grabbing of the steering wheel. In either case, the passenger's actions rather than the driver's would be considered the proximate cause of the pedestrian's injuries.

A boy was playing softball in a neighborhood park when a ball was hit over the fence and into a neighbor's yard. The boy knocked on the neighbor's door and obtained permission from her to retrieve the ball from her yard. As he bent to retrieve the ball in some bushes, the boy brushed against an exposed electric wire that was partially hidden by the bushes and received a severe electric shock and burns. The neighbor had failed to maintain the bushes, allowing them to become overgrown, and was not aware of the exposed wire. If the boy sues the neighbor in a jurisdiction that applies the traditional rules for landowners and possessors of land, what is the likely result? response - incorrect A The neighbor is liable because the boy entered with her permission. B The neighbor is liable because she failed to repair a dangerous condition on her property. C The neighbor is liable because she failed to reasonably inspect the property, which would have made her aware of the dangerous condition of the wire. D The neighbor is not liable because she did not know of the condition of the wire.

Answer: D (mine: B) Explanation: The neighbor is not liable because she did not know of the condition of the wire and the boy was a licensee. In jurisdictions that distinguish between invitees and licensees, a licensee is a person who enters land with the owner's permission, for his own purpose or business rather than for the owner's benefit. The owner or occupier of land owes the licensee a duty to warn of or make safe a dangerous condition known to the owner or occupier that creates an unreasonable risk of harm to the licensee and that the licensee is unlikely to discover. However, the owner or occupier has no duty to a licensee to inspect for defects nor to repair known defects. The boy was a licensee because he entered the neighbor's land with her permission for his own purpose (retrieving the ball) rather than for any benefit of the neighbor's. The exposed electric wire created an unreasonable risk of death or serious injury to the boy as he reached into the bushes. Because the neighbor did not know of the presence and condition of the wire, she had no knowledge of any risk of harm to the boy. Thus, no duty to warn the boy of the wire was triggered. (A) is incorrect because the mere fact that the neighbor gave the boy permission to enter her land will not subject her to liability for his injuries incurred thereon. The neighbor is not strictly liable for injuries to a licensee, but only for any injuries caused by a breach of her duty to warn of dangerous conditions known to her and that the licensee is unlikely to discover. (A) would impose liability even where the neighbor had no knowledge of the condition of the wire. (B) is incorrect because, as noted above, an owner of land does not owe a duty to a licensee to repair defects or dangerous conditions. Likewise, (C) is incorrect because the owner of land is not under a duty to a licensee to make an inspection to discover defects or dangerous conditions.

In a products liability action based on strict liability, the plaintiff need show only (i) the defendant is a __________ ________, (ii) the defendant _________ or ____ a _________ product, (iii) the ________ _______ was an actual and proximate cause of the plaintiff's injury, and (iv) the plaintiff _______ _______ to ______ or ________.

COMMERCIAL SUPPLIER PRODUCE or SOLD a DEFECTIVE product DEFECTIVE PRODUCT SUFFERED DAMAGES to PERSON or PROPERTY

Failure to follow industry custom raises a presumption of negligence. True or false?

False.

If more than one person may have been in control of the instrumentality that caused the injury, most courts will require each defendant to prove that he or she was not negligent. True or false?

False.

When a custom, standard, or usage of an industry is relevant, it replaces the general negligence standard. True or false?

False.

In Palsgraf, J. Cardozo opined that under tort law the duty of care is owed only to [_______] plaintiffs----those within the zone of danger

Foreseeable

4 elements of res ipsa

I) Accident causing injury = type that would not ordinarily occur unless someone was negligent (e.g., product malfunction, something falling out of sky) II) Negligence attributable to D (e.g., happened on D's premises, was D's product, was D's employees III) P was NOT contributorily negligent. IV) Instrumentality was w/in the exclusive control of the D (i.e. only one w/access to service product or maintain escalator handrail)

Is a hiker hiking on the landowner's open land with permission an invitee?

No. A hiker on the landowner's open land is not considered an invitee. If an owner or occupier of open land permits the public to use the land for recreational purposes without charging a fee, the landowner is not liable for injuries suffered by a recreational user unless the landowner willfully and maliciously failed to guard against or warn of a dangerous condition or activity.

The general duty of care under negligence requires a person to act as an ordinary, [Reasonable/prudent] person would under the circumstances

Prudent

In proving actual cause in a product-liability case based on strict liability, what is the actual-cause requirement that a Plaintiff must show & what 2 benefits are Ps allotted when seeking to show said requirement?

Requirement: The defect in the product must have existed when the product left D's control Benefit 1: If the defect is difficult to trace, P may rely on an inference that such a product failure would ordinarily occur only as a result of the defect. Benefit 2: If the defect is FTW, then P is entitled to a presumption that if an adequate warning had been included, then P would've read and heeded such a warning.

What are the elements of the one superseding cause that defendant-M can rely upon in regards to the co-defendant-retailer's conduct to sever M's liability to buyer-plaintiff?

That retailer both (I) actually discovered the defect during the course of an inspection & (II) failed to warn the buyer-plaintiff of said defect.

QUESTION Three months ago, a furniture store's delivery driver delivered furniture to a longtime customer. In violation of a local traffic ordinance, the driver double-parked the delivery van in front of the customer's house. A few minutes later, while the driver was in the house, a car drove around the delivery van into the oncoming lane. The car was struck by a garbage truck that was hidden from view by the delivery van, and a passenger in the car was seriously injured. The passenger has brought a negligence action against the store to recover damages for injuries resulting from the driver's conduct. Pretrial discovery has revealed that delivery vans routinely double-park when parking is not available; survey evidence suggests that, in urban areas like this one, 80% of deliveries are made while the delivery van is double-parked. The store moves for summary judgment, setting forth the above, undisputed facts and arguing that the driver could not be found liable for the passenger's injuries. How should the court rule on the motion? ANSWER The driver may be found negligent on these facts, so the motion for summary judgment should not be granted. The key issue here is whether following the industry custom of double-parking negates a finding of negligence. A defendant will be liable in a negligence cause of action if the plaintiff can prove that the defendant owed the plaintiff a duty, the defendant breached that duty, the defendant's actions were the actual and proximate cause of the plaintiff's injuries, and the plaintiff incurred damages. At a minimum, a person owes a duty to others to act as an ordinary, prudent person would act under like circumstances. However, where the conduct in question is in violation of a statute, proof of the violation will constitute negligence per se, conclusively establishing the duty owed and the breach of duty, if the plaintiff can establish three things: (i) a clearly stated specific duty, (ii) the plaintiff was within the class of people intended to be protected by the statute, and (iii) the harm suffered by the plaintiff was the type of harm the statute was designed to prevent. Absent an applicable statutory standard, industry custom can provide some evidence of whether a duty is breached, but such evidence is not conclusive. [A] Moreover, a local ordinance specifically prohibits double-parking. Although we have no statutory history to determine the purpose of the ordinance, it was no doubt intended, at least in part, to prevent traffic accidents (e.g., involving vehicles going into oncoming traffic to get around the double-parked vehicle). [B] The fact that the delivery driver was following industry custom in double-parking (i.e., delivery vans routinely double-park when parking is not available and 80% of deliveries are made while the delivery van is double-parked) does nothing to negate the driver's negligence. Arguably, following industry custom shows that the driver was doing what drivers ordinarily do, and what people ordinarily do usually is not a breach of duty. However, following industry custom does not raise a presumption that there was no breach of duty. [C] Here, the delivery driver double-parked his delivery van which, arguably, is a breach of the duty of ordinary care. The driver's double parking caused the car in which the passenger was riding to steer into the oncoming lane. [A] Moreover, a local ordinance specifically prohibits double-parking. Although we have no statutory history to determine the purpose of the ordinance, it was no doubt intended, at least in part, to prevent traffic accidents (e.g., involving vehicles going into oncoming traffic to get around the double-parked vehicle). [D] Therefore, the driver's following custom does nothing to disprove the breach of duty here, so the motion for summary judgment should not be granted. [E] More importantly, a statutory standard prohibiting double-parking applies here, and the industry custom cannot negate the law. [F] The car was hit by a garbage truck that was hidden by the delivery van, injuring the passenger and satisfying the prima facie case for negligence. [G] The plaintiff is surely in the class of people sought to be protected, since the plaintiff was injured when the car she was in steered into the oncoming lane to get around the double-parked delivery van.

[C] Here, the delivery driver double-parked his delivery van which, arguably, is a breach of the duty of ordinary care. The driver's double parking caused the car in which the passenger was riding to steer into the oncoming lane. [A] Moreover, a local ordinance specifically prohibits double-parking. Although we have no statutory history to determine the purpose of the ordinance, it was no doubt intended, at least in part, to prevent traffic accidents (e.g., involving vehicles going into oncoming traffic to get around the double-parked vehicle). [F] The car was hit by a garbage truck that was hidden by the delivery van, injuring the passenger and satisfying the prima facie case for negligence. [A] Moreover, a local ordinance specifically prohibits double-parking. Although we have no statutory history to determine the purpose of the ordinance, it was no doubt intended, at least in part, to prevent traffic accidents (e.g., involving vehicles going into oncoming traffic to get around the double-parked vehicle). [G] The plaintiff is surely in the class of people sought to be protected, since the plaintiff was injured when the car she was in steered into the oncoming lane to get around the double-parked delivery van. [B] The fact that the delivery driver was following industry custom in double-parking (i.e., delivery vans routinely double-park when parking is not available and 80% of deliveries are made while the delivery van is double-parked) does nothing to negate the driver's negligence. Arguably, following industry custom shows that the driver was doing what drivers ordinarily do, and what people ordinarily do usually is not a breach of duty. However, following industry custom does not raise a presumption that there was no breach of duty. [E] More importantly, a statutory standard prohibiting double-parking applies here, and the industry custom cannot negate the law. [D] Therefore, the driver's following custom does nothing to disprove the breach of duty here, so the motion for summary judgment should not be granted.

A statute prohibits drivers from leaving their cars running. A driver exits his car with the engine running. When the driver slams his door, the car slips into gear and runs over the plaintiff. The plaintiff probably can use the statutory standard in place of the general standard to prove duty and breach. True or false?

True.

In a products liability case based on strict liability, which way does indemnification flow?

Upstream only; i.e. Retailer can be indemnified by M; M CANNOT be indemnified by retailer.

In cases where a force came into motion after the time of the defendant's negligent act and combined with the negligent act to cause injury to the plaintiff, this intervening force will likely be foreseeable under what circumstances?

Where the defendant's tortious conduct increased the risk that this force would cause harm to the plaintiff.

In Palsgraf, J. Andrews opined that under tort law the duty of care is owed to _________ who suffers an injury as a proximate result of D's breach to someone

anyone

If a statute has a clearly stated _________ duty, that duty can _________ replace the more general common law duty of care if P also shows P is within the _________ of people intended to be protected by the statute and the harm suffered was the type of harm that the statute was designed to prevent.

specific replace class


Ensembles d'études connexes

Chapter 14, Chapter 13, Chapter 12, Chapter 2

View Set

Security + 501 Chapter 3 Architecture and Design

View Set

Chapter 7 Quiz: Individual and Group Decision Making

View Set

5.1 Conscious and Unconscious: The Mind's Eye, Open and Closed

View Set

Ch 17: Mental Health Promotion for Young and Middle-Aged Adults

View Set